Đến nội dung

xuantrandong nội dung

Có 45 mục bởi xuantrandong (Tìm giới hạn từ 25-04-2020)



Sắp theo                Sắp xếp  

#636982 VMF's Marathon Hình học Olympic

Đã gửi bởi xuantrandong on 31-05-2016 - 00:03 trong Hình học

$\boxed{\text{Lời giải bài toán 17}}$

Gọi $X$ và $Y$ là giao điểm của $EF$ và đường tròn ngoại tiếp tam giác $ABC$

xét phép nghịch đảo cực $A$ phương tích $AX^{2}=AY^{2}=AF.AB=AE.AC$ ta có đường tròn $(A;AX)$ trực giao với đường tròn đường kính $BC$ hay$P_{M,(A;AX)}=MB^{2}=MC^{2}$ 

Gọi $K$ là giao điểm $EF$ và $BC$

Gọi $U$ và $V$ là giao điểm của $MX,MY$ và  $(A;AX)$

Do $(K,D,B,C)=-1$ nên theo $Maclaurin$ ta có $KD.KM=KB.KC=KX.KY$ suy ra $XYMD$ nội tiếp $\Rightarrow \angle DXU=\angle DYV$ 

Do $(K,D,B,C)=-1$ nên theo $Newton$ ta có $MX.MU=MB^{2}=MD.MK$ suy ra $XUDK$ nội tiếp

tương tự $YVDK$ nội tiếp

ta có $\angle DKU=\angle DXU=\angle DYV=\angle DKV$ nên $K,U,V$ thẳng hàng

mà $MF^{2}=MD.MK$ nên $\angle DFM=\angle DKF=\angle DUM$ nên $DMFU$ nội tiếp tương tự $DMEV$ nội tiếp

$\Rightarrow $ đường thằng chứa $K,U,V$ chính là trục đẳng phương của đường tròn $Euler$ và   $(A;AX)$

ta có tâm đường tròn $Euler$, tâm $A'$ của $(BHC)$ và $A$ thằng hàng  nên $AA'$ vuông góc  $UV$

mà $KX.KY=KB.KC= KU.KV$ và $K$ thuộc đương thẳng $UV$ nên $(A;AX)$, đường tròn $Euler$ và đường tròn $(BHC)$ có 2 điểm chung là $U,V$

vậy  $U\equiv P,V\equiv Q$

Áp dụng định lý $Brocard$ cho tứ giác $BECF$ nội tiếp ta có $H$ là trực tâm của tam giác $AKM$, mà $A$ là tâm  $(A;AX)$  nên theo định lý $Brocard$ cho tứ giác nội tiếp $XYPQ$ thì $H$ là giao điểm $XQ,YP\Rightarrow XQ$ đi qua $H$

vậy $MP,QH,EF$ đồng quy tại $X$ ta có đpcm.

Untitled.jpg




#637113 VMF's Marathon Hình học Olympic

Đã gửi bởi xuantrandong on 31-05-2016 - 13:35 trong Hình học

Mình xin đề xuất $\boxed{\text{Bài toán 18}}$

(Sưu tầm) Cho$\Delta ABC$ nhọn. $M$ là một điểm di động trên cạnh $AB$ và $N$ là trung điểm $AC$. Gọi $P,Q$ là hình chiếu của $A$ trên $MC,MN$. Chứng minh rằng khi $M$ di động trên cạnh $AB$ thì tâm đường tròn ngoại tiếp $\Delta PQN$ nằm trên một đường thẳng cố định.




#636005 VMF's Marathon Hình học Olympic

Đã gửi bởi xuantrandong on 27-05-2016 - 18:01 trong Hình học

Xin lỗi thầy và mọi người, bài toán vừa rồi mình lấy trong tài liệu 111 nice geometry problems của diễn đàn Mathscope. không ngờ lại trùng bài viết của thầy Hùng

Mình xin up lại bài khác:

$\boxed{\text{Bài toán 9}}$

Cho $\triangle ABC$ có trực tâm $H$. $P,Q$ bất kì thuộc cạnh $AB,AC$ dựng điểm M sao cho $\triangle MBC$ đồng dạng $\triangle HPQ$ và $M$ cùng phía với $A$ đối với cạnh $BC$. Chứng minh rằng $MH$ vuông góc $PQ$




#635957 VMF's Marathon Hình học Olympic

Đã gửi bởi xuantrandong on 27-05-2016 - 15:42 trong Hình học

$\boxed{\text{Lời giải bài toán 8}}$ 

 

Gọi $N,M$ là giao điểm của $O_{1}O_{2}$ với $ O_{1}$ và $O_{2}$ như hình trên, $H$ là giao điểm thứ 2 của $O_{1}P$ và  $(O_{2})$, $G$ là giao điểm thứ 2 của  $O_{2}P$ và  $(O_{1})$

ta sẽ chứng minh $EM,FN$ và $AB$ đồng quy

ta có $(O_{2}PFG)=(O_{1}PEH)=-1$ nên $EF, GH, O_{1}O_{2}$ đồng quy tại $Y$

Gọi $I$ là điểm sao cho $(IPAB)=-1$ thì $IF,IG$ là tiếp tuyến của $O_{1}$  và $IE,IH$ là tiếp tuyến của $O_{2}$, $I$ thuộc $AB$ là trục đẳng phương của 2 đường tròn nên  $IE=IF=IG=IH$ nên $EFGH$ nội tiếp 

suy ra $EG,FH,O_{1}O_{2}$ đồng quy tại điểm $X$ ( theo định lý Brocard) và $(YXO_{1}O_{2})=-1$ nên $PX, O_{1}F, O_{2}E$ đồng quy.

Áp dụng định lí Ceva để ý $R_{1}^{2}=O_{1}E.O_{1}H $ và $R_{2}^{2}=O_{2}F.O_{2}G$  ta có $\frac{XO_{1}}{XO_{2}}=\frac{O_{1}E}{EP}.\frac{PF}{FO_{2}}=\frac{O_{1}H}{PH}.\frac{GP}{GO_{2}}=\frac{R_{1}^{2}}{R_{2}^{2}}.\frac{GP.O_{2}F}{PH.O_{1}E}=\frac{R_{1}^{2}}{R_{2}^{2}}.\frac{EP.O_{2}F}{FP.O_{1}E}=\frac{R_{1}^{2}}{R_{2}^{2}}.\frac{O_{2}X}{O_{1}X}$

nên $\frac{O_{1}X}{O_{2}X}=\frac{R_{1}}{R_{2}}$ nên $X$ là tâm vị tự trong của 2 đường tròn $\Rightarrow Y$ là tâm vị tự ngoài của 2 đường tròn do $(YXO_{1}O_{2})=-1$ $\Rightarrow $ theo tính chất cơ bản ta có $EFMN$ nội tiếp $\Rightarrow EM,FN,AB$ đồng quy tại tâm đẳng phương $V$ của 3 đường tròn

dễ thấy $EM,FN$  là phân giác $\angle PED$ và  $\angle PFD$ nên $EV, FV$ cũng là phân giác 2 góc này

Từ định lý phân giác ta có điều phải chứng minh.

Post 167.png

$$\begin{array}{| l | l |} \hline Ngockhanh99k48 & 1\\ \hline IHateMath & 1\\ \hline fatcat12345 & 2\\ \hline dogsteven & 2\\ \hline baopbc & 3\\ \hline QuangDuong12011998 & 1\\ \hline xuantrandong & 1\\ \hline\end{array}$$




#631733 ĐỀ THI OLYMPIC CHUYÊN KHOA HỌC TỰ NHIÊN 2016

Đã gửi bởi xuantrandong on 07-05-2016 - 13:01 trong Thi HSG cấp Tỉnh, Thành phố. Olympic 30-4. Đề thi và kiểm tra đội tuyển các cấp.

câu hình ý a khá đơn giản sử dụng $\Delta FIN$ đồng dạng $\Delta IEM$

câu hình b gọi T và H là trung điểm EF và EI, ta có $\Delta FIE$ đồng dạng $\Delta IME$ nên $\Delta FIT$ đồng dạng $\Delta IMH$ nên $\angle FIT = \angle IMH=\angle = \angle MIC =\frac{\angle B}{2}$ nên TI là tiếp tuyến của (L) $=>$  TI là trục đẳng phương của (L) và (K) mà T là trung điểm EF nên dễ suy ra được $EP= FQ$




#631938 ĐỀ THI OLYMPIC CHUYÊN KHOA HỌC TỰ NHIÊN 2016

Đã gửi bởi xuantrandong on 08-05-2016 - 14:43 trong Thi HSG cấp Tỉnh, Thành phố. Olympic 30-4. Đề thi và kiểm tra đội tuyển các cấp.

L thuộc đường đối cực của B và M nên BM là đường đối cực của L

=> IL vuông góc BM 

tương tự IK vuông góc CN

ta có đpcm




#629302 Đề thi học sinh giỏi môn toán khối 10 khu vực DUYÊN HẢI VÀ ĐỒNG BẰNG BẮC BỘ n...

Đã gửi bởi xuantrandong on 24-04-2016 - 11:38 trong Thi HSG cấp Tỉnh, Thành phố. Olympic 30-4. Đề thi và kiểm tra đội tuyển các cấp.

giai thich cho minh cho nay dc k

 

 

Gọi $A_1$ là phần tử có nhiều nhất trong $64$ tập con và $a_1$ là số tập con chứa $A_1$, ta có:

$a_1\geq \frac{64.1008}{2016}=32$




#679338 a + b - (c * a) = d Biết b,c,d Tìm a (Hỏi công thức)

Đã gửi bởi xuantrandong on 03-05-2017 - 16:23 trong Số học

VD: a + b - (c * a) = d

a + 11 - (20 * a) = -502 Có bạn nào biết công thức để tìm a ở bài toán này không chỉ mình với _thank.




#679341 a + b - (c * a) = d Biết b,c,d Tìm a (Hỏi công thức)

Đã gửi bởi xuantrandong on 03-05-2017 - 16:37 trong Số học

Cảm ơn bạn nhiều nhá có công thức cái học và làm bài rễ hẳn. :D




#679704 a + b - (c * a) = d Biết b,c,d Tìm a (Hỏi công thức)

Đã gửi bởi xuantrandong on 06-05-2017 - 16:40 trong Số học

Nếu $c*a=c.a$ thì công thức trên đúng rồi còn kết quả trên máy tính nó tính lệch thôi, đúng đến chữ số thập phân số 9 mà. Số càng lớn thì máy tính tính càng sai số. Với lại công thức này thực sự rất dễ suy ra, chẳng qua nhóm hạng tử $a$ ra thôi.

P/s: chắc bạn chuyên lí.

:D Thank bạn, máy tính của mình nó bị sao ý 2,7 - 1 nó lại ra kết quả là 1,7000000000000002 có lẽ mấy mụ đống nát đã yểm bùa nó rồi.




#679665 a + b - (c * a) = d Biết b,c,d Tìm a (Hỏi công thức)

Đã gửi bởi xuantrandong on 06-05-2017 - 10:42 trong Số học

(c * a) chính là (c nhân a) trên máy tính dấu * chính là dấu nhân.




#679611 a + b - (c * a) = d Biết b,c,d Tìm a (Hỏi công thức)

Đã gửi bởi xuantrandong on 05-05-2017 - 20:08 trong Số học

Cảm ơn bạn đã quan tâm tới bài toán của mình.
a,b,c là số ngẫu nhiên. Bài đó NHoang1608 giải được rồi. Tuy công thức của NHoang1608 giải nhiều lúc sai vài số cuối :D.
VD:
a = 207483.7826339174
b = 39445.630410252605
c = 750998.976752332
d = a + b - (c * a) = -155819861521.36218
 
a = 207483.78263391738 << Là kết quả tính a theo công thức (d-b)/(1-c) của NHoang1608
a = 207483.782633917474 << Nhưng kết quả đúng đáng nhẽ phải ra thế này



#679664 a + b - (c * a) = d Biết b,c,d Tìm a (Hỏi công thức)

Đã gửi bởi xuantrandong on 06-05-2017 - 10:39 trong Số học

b = 39445.630410252605
c = 750998.976752332
d = 155819861521.36218
a = ?
 
Tìm a sao cho a + b - (c * a) = d
 
Chỉ là một bài toán tìm a thôi mà có cần phải quan tâm tới c * a nhiều đến vậy không?



#684944 Tuần 3 tháng 6/2017: Chứng minh rằng $\angle ADO= \angle OAG...

Đã gửi bởi xuantrandong on 18-06-2017 - 21:22 trong Chuyên mục Mỗi tuần một bài toán Hình học

Bài 2

Hình gửi kèm

  • 1.png



#678530 Tuần 4 tháng 4/2017: Đường tròn pedal của $A$ ứng với tam giác...

Đã gửi bởi xuantrandong on 24-04-2017 - 22:01 trong Chuyên mục Mỗi tuần một bài toán Hình học

Bài 1:

Gọi $D, E, F$ là điểm đối xứng của $A$ qua $KL, KN, LM$, ta cần chứng minh $(DEF)$ tiếp xúc $BC$.

Gọi $AP$ là đường đối trung của tam giác $ABC$ với $P$ thuộc $(ABC)$

Thấy rằng $D$ là giao điểm của đường tròn qua $A,B$ tiếp xúc $AC$ và đường tròn qua $A,C$ tiếp xúc $AB$

                 $E$ là giao điểm của đường tròn qua $A,C$ tiếp xúc $BC$ và đường tròn qua $A,P$ tiếp xúc $AC$

                 $F$ là giao điểm của đường tròn qua $A,B$ tiếp xúc $BC$ và đường tròn qua $A,P$ tiếp xúc $AB$

Sử dụng phép hợp phép nghịch đảo cực $A$ phương tích $AB.AC$ và phép đối xứng qua phân giác góc $\angle A$ ta được bài toán sau:

Cho $\Delta ABC$ nội tiếp $(O)$, tiếp tuyến tại $B,C$ giao nhau tại $T$, $M$ là trung điểm BC. $H,G$ thuộc $TC,TB$ sao cho $MH // AC$ và $MG // AB$ . Chứng minh rằng $THG$ tiếp xúc $(O)$.

Chứng minh:

$AM,AT$ giao $(O)$ tại $F,I$. kẻ tiếp tuyến $Ix$ của $(O)$.

Dễ chứng minh $BMIG,CMIH,GTHI$ là các tứ giác nội tiếp

Biến đổi góc như sau ( để ý $IF//BC$)

$\angle HIx=\angle CIH-\angle CIx=\angle HMC-\angle FBT=\angle ACB-\angle FBT=\angle AFB-\angle FBT=\angle BTF=\angle HTI$

Do đó $Ix$ cũng là tiếp tuyến của $THG$, do đó $THG$ tiếp xúc $(O)$.

Hình gửi kèm

  • images.png



#678449 Tuần 4 tháng 4/2017: Đường tròn pedal của $A$ ứng với tam giác...

Đã gửi bởi xuantrandong on 23-04-2017 - 21:57 trong Chuyên mục Mỗi tuần một bài toán Hình học

Mình xin phép giải bài 2:

Chuyển mô hình bài toán về bài toán sau đây:

Cho $\Delta ABC$ nội tiếp đường tròn $(O)$ có $H$ là trực tâm. 3 đường cao $AD, BE, CF$. Đường thẳng $HO$ cắt $(FHD)$ tại $V$. $Y$ là điểm thuộc $(FHD)$ sao cho $HY$ vuông góc $FV$. Chứng minh rằng điểm đối xứng của $Y$ qua $HO$ thuộc đường tròn $(DHE)$. 

Giải:

Gọi $K, L$ đối xứng với $F, E$ qua $BE, CF$. $KL$ giao $(FHD)$ và $(EHD)$ lần lượt tại $Y', X'$. Ta sẽ chưng minh $OH$ là đường trung trực của $X'Y'$ và $HY'$ vuông góc $FV$, và từ đó ta chứng minh được $X$ trùng $X'$ và $Y$ trùng $Y'$.

Gọi $B', C'$ đối xứng với $B, C$ qua $CF, BE$. $(I)$ là đường tròn ngoại tiếp $\Delta HB'C'$. Theo $APMO-2010$ thì $I, H, O$ thẳng hàng. Dễ thấy theo phép đối xứng trục thì $B', H, L$ và $C', H, K$ thẳng hàng. và $\overline{HK}.\overline{HC'}=\overline{HF}.\overline{HC}=\overline{HE}.\overline{HB}=\overline{HL}.\overline{HB'}$ do đó theo phép nghịch đảo tâm $H$ phương tích $\overline{HA}.\overline{HD}$ thì $IH$ vuông góc $LK$ tức là $X'Y'$ vuông góc $OH$. Gọi $M, N$ là giao điểm $(I)$ và $AC, AB$ thì theo phép nghịch đảo tâm $H$ phương tích $\overline{HA}.\overline{HD}$ thì $CC' \leftrightarrow (FHD) $, $(I) \leftrightarrow X'Y'$ do đó $M \leftrightarrow Y'$, tương tự $N \leftrightarrow X'$. mà $\angle HC'M=\angle HCA=\angle HBA=\angle HB'N$ nên $HM = HN$. Do đó $HX'=HY'$. mà $OH$ vuông góc $X'Y'$ nên $OH$ là đường trung trực $X'Y'$.

Ta có $\angle HVF+\angle VHY'=\angle HBA+\angle IHM=\angle HC'M+\angle IHM=90$ nên $FV$ vuông góc $HY'$

Do đó $X$ trùng $X'$, $Y$ trùng $Y'$ nên ta có điều phải chứng minh

Hình gửi kèm

  • images.png



#682327 Tuần 5 tháng 5/2017: Chứng minh rằng bốn điểm $R,H,J,K$ cùng thuộc...

Đã gửi bởi xuantrandong on 29-05-2017 - 19:14 trong Chuyên mục Mỗi tuần một bài toán Hình học

Lời giải bài 2 của mình

Bổ đề: Cho $\Delta ABC$, 2 điểm $P,Q$ sao cho $P$ nằm ở miền trong $\Delta ABC$ và $BPCQ$ là hình bình hành. khi đó $BP,CP$ đối song $\angle A$ khi và chỉ khi $AP,AQ$ đẳng giác $\angle A$

Chứng minh: Dựng hình bình hành $ACPR$

Giả sử $BP,CP$ đối song thì $\angle ABP=\angle ACP=\angle ARP$ khi đó $ARBP$ nội tiếp nên $\angle BAP=\angle BRP=\angle CAQ$ do đó $AP,AQ$ đẳng giác. 

Chiều ngược lại chứng minh tương tự 

Trở lại bài toán: 

Lấy $P,Q$ đối xứng với $N,M$ qua trung điểm $L$ của $BC$. Áp dụng bổ đề:

Do $BM,CM$ đối song góc $A$ nên $AM,AQ$ đẳng giác góc $A$.

Do $BP,CP$ đối song góc $A$ nên $AN,AP$ đẳng giác góc $A$. 

vậy nên $AM,AQ$ đẳng giác $\angle PAN$, áp dụng bổ đề 1 lần nữa được $PM,NM$ đối song góc $\angle PAN$.

Dựng hình bình hành $AH'MP$ được $H'AMN$ nội tiếp do $\angle AH'M=\angle APM=\angle ANM$

và $AH'$ vuông góc $BC$ nên $H$ trùng $H'$.

Vậy $AH=MP=2KL$

Hình gửi kèm

  • 1.png



#677778 Tuần 3 tháng 4/2017: Chứng minh rằng đường thẳng $QR$ đi qua điểm c...

Đã gửi bởi xuantrandong on 17-04-2017 - 21:38 trong Chuyên mục Mỗi tuần một bài toán Hình học

Mình xin phép giải bài 2:

Ta chuyển bài toán trên thành bài toán tương đương như sau: Cho tam giác ABC, P di động trên BC, đường thẳng qua P vuông góc AB, AC cắt các đường thẳng AC, AB tại E, F. Chứng minh rằng tâm đường tròn ngoại tiếp tam giác PEF thuộc một đường thẳng cố đinh.

Đường thẳng qua B và C lần lượt vuông góc AB và AC cắt AC và AB tại M, N. R, S là trung điểm AM, AN. L, K là trung điểm BM, CN. X, Y đối xứng R, S qua L, K. H, I là trung điểm PE, PF. 

Dễ thấy ACYS và ABXR là hình bình hành nên SY // BX // AC và RX // CY // AB. Theo bổ đề hình thang thì B, I, K thẳng hàng và C, H, L thẳng hàng. Qua I kẻ đường thẳng song song với AC giao XY tại O thì áp dụng liên tiếp định lý talet:

$\frac{YO}{OX}=\frac{KI}{IB}=\frac{CP}{PB}=\frac{CH}{HL}$

mà CY // LX nên HO // CY // LX // AB

Vậy O là tâm đường tròn ngoại tiếp tam giác PEF di động trên đường thẳng XY cố định

Hình gửi kèm

  • images.png



#629452 Chứng minh $KD$ là phân giác $\angle IKH$

Đã gửi bởi xuantrandong on 24-04-2016 - 23:06 trong Hình học

Cho tam giác $ABC$ có $I$ là tâm nội tiếp. $(I)$ tiếp xúc với $BC,CA,AB$ tại $D,E,F.$ Hạ $DK\perp EF$ và $H$ là trực tâm tam giác $ABC$.

Chứng minh $KD$ là phân giác $\angle IKH$.

 

Mình sử dụng $Anti steiner$ và vị tự nhưng thấy khá phức tạp! Mọi người ai có ý kiến gì thì đóng góp nhé!




#629983 Chứng minh $KD$ là phân giác $\angle IKH$

Đã gửi bởi xuantrandong on 28-04-2016 - 12:41 trong Hình học

Mình cũng sử dụng bổ đề $2$ của bạn, mình chứng minh nó bằng phép vị tự. 

Kẻ đường kính $AM$. 

Để ý nếu đường tròn ngoại tiếp tam giác $AEF$ cắt $\odot (O)$ tại điểm $X$ thì theo bổ đề $2$: $X,K,I,M$ thẳng hàng tức là $XM,EF$ cắt nhau tại $K$. 

Lấy $I'$ đối xứng với $I$ qua $EF$ thì $I'$ là trực tâm tam giác $AEF$.

Áp dụng định lí về điểm $Anti steiner$ dễ chứng minh được $X$ chính là điểm $Anti steiner$ đối với đường thẳng $HI'$ của tam giác $AEF$ nên $HI',EF$ và $XM$ từ đó nhận thấy $HI',EF$ và $XM$ đồng quy tại $K$. 

Do 2 đường thẳng $HI'$ và $XM$ đối xứng nhau qua $EF$ (theo $Anti steiner$) nên $KD $là phân giác $\angle IKH$

Bài toán được chứng minh.




#680738 Tuần 3 tháng 5/2017: đường thẳng $AQ$ luôn đi qua một điểm cố định...

Đã gửi bởi xuantrandong on 15-05-2017 - 03:26 trong Chuyên mục Mỗi tuần một bài toán Hình học

Lời giải bài $2$ của mình dài và chưa hay  

Bổ đề: Trên đường thẳng $d$, lấy các điểm $X,Y,E,F,M,N,I$ sao cho $(XYFE)=-1$, $I$ là trung điểm $EF$, $\overline{YM}.\overline{YN}=\overline{YX}.\overline{YI}$. Khi đó ta có $(MFIX)=(NEXI)$

Chứng minh bổ đề: Gọi tọa độ các điểm $X,E,F,M,N,I$ trên đường thẳng $d$ là $x,e,f,m,n,i$, chọn $Y$ làm gốc tọa độ.

Ta có (dễ thấy) :  $i=\frac{e+f}{2}$

                           $mn=x(\frac{e+f}{2})$

                           $x=\frac{2ef}{e+f}$

Biến đổi: $(MFIX)=(NEXI)\Leftrightarrow \overline{IM}.\overline{IN}.\overline{XE}.\overline{XF}=\overline{XM}.\overline{XN}.\overline{IE}.\overline{IF}\Leftrightarrow (m-\frac{e+f}{2})(n-\frac{e+f}{2})(x-e)(x-f)=-(x-m)(x-n)\frac{(e-f)^{2}}{4}\Leftrightarrow (\frac{(e+f)^{2}}{4}-\frac{(m+n)(e+f)}{2}+mn)=-(x^{2}-x(m+n)+mn)(\frac{(e-f)^{2}}{4})$

Thay  $mn=x(\frac{e+f}{2})$ vào ta có $\frac{e+f}{2}.(x-e)(x-f)=-x.\frac{(e-f)^{2}}{4}$, tiếp tục thay  $x=\frac{2ef}{e+f}$ vào ta được đẳng thức cuối cùng đúng, suy ra điều phải chứng minh.

Trở lại bài toán

Gọi $K$ là trung điểm $AH$, $I$ là trung điểm $EF$, đường thẳng $EF$ giao $BC,AH$ tại $X,Y$, đường thẳng qua $K$ vuông góc $BK, CK$ giao $AC,AB$ tại $U,V$.

Dễ thấy $BKEU$ là tứ giác nội tiếp, nên $\angle KBU=\angle KEA=\angle KAE=\angle HBC\Rightarrow \angle KBE=\angle UBC$

Mà $\angle ABH=\angle OBC$ nên $\angle ABK=\angle OBU$, do đó $O,K$ đẳng giác trong $\Delta ABU$, nên $\angle OUB=\angle KUA=\angle KBE=\angle UBC$ do đó $OU$ song song $BC$. Mặt khác hai tam giác $BHA,BFE$ đồng dạng có $K,I$ là trung điểm của 2 cạnh tương ứng $HA,FE$ nên $\angle KBE=\angle IBA$ nên $\angle UBC=\angle IBA$

Chứng minh tương tự ta cũng được $\angle ICA=\angle VCB$

Do $OU,OV$ song song $BC$ nên $P,U,O,V$ thẳng hàng, nên $B(CUPV)=C(BUPV)$, xét phép đối xứng qua phân giác góc $B$ và phép chiếu xuyên tâm $B$ lên đường thẳng $EF$ thì $B(CUPV)=B(AIQC)=(FIMX)$, tương tự $C(BUPV)=C(ABQI)=(EXNI)$

Do đó $(FIMX)=(EXNI) \Rightarrow (MFIX) = (NEXI )$

Gọi đường tròn $(AMH)$ giao $EF$ tại $N'$ thì theo bổ đề ta có $(MFIX)=(N'EXI)$, vậy $N$ trùng $N'$

Dễ thấy $(XYFE)=-1$ do đó $(YXFE)=-1$, áp dụng hệ thức $Maclaurin$ thì $\overline{YI}.\overline{YX}=\overline{YE}.\overline{YF}$

Vậy $\overline{YM}.\overline{YN}=\overline{YX}.\overline{YI}=\overline{YH}.\overline{YA}$ nên $A,H,M,N$ cùng thuộc 1 đường tròn.

Hình gửi kèm

  • 1.png



#632950 Chứng minh $P,B,L,S$ đồng viên

Đã gửi bởi xuantrandong on 13-05-2016 - 21:23 trong Hình học

Đây là lời giải của mình:

Gọi $R$ là giao điểm $HL$ và $BC$. Qua $A$ kẻ đường thằng song song với $BC$ cắt $EF$ và $HL$ tại $Q$ và $U$.

Dễ thẩy $R, E, F, Q$ thằng hàng mà chùm $R(A,H,F,B)=-1$ nên $Q$ là trung điểm $AU$ nên $PQ$ là đường trung trực của $AL$.

$QURP$ là hình bình hành mà $AQ=QU$ nên $AQRP$ là hình bình hành nên $\angle AQF=\angle APC$ nên $\triangle AQF$ đồng dạng $\triangle APC$ mà $\triangle AEF$ đồng dạng $\triangle ABC$ nên $\frac{QE}{EF}=\frac{PB}{BC}$.

Dế chứng minh được $\triangle LEF$ đồng dạng $\triangle LBC$ nên kết hợp điều trên ta có $\Delta LQE$ đồng dạng $\Delta LPB$ suy ra $\Delta LEB$ đồng dạng $\Delta LQP$ nên $LEFQ$ nội tiếp $\Rightarrow \angle LSE=\angle LQE$ nên $\angle LSB=\angle LQR$  

Dễ thấy $LQPR$ là hình thang cân $\Rightarrow \angle LQR=\angle LPR$ vậy $\angle LSB=\angle LPB$ ta có điều phải chứng minh.

Untitled.png

P/s: Mình làm hơi tắt, mấy bạn thông cảm




#630516 Chứng minh $AH$ và $AI$ đẳng giác

Đã gửi bởi xuantrandong on 01-05-2016 - 12:35 trong Hình học

Cho tam giác $ABC$ nhọn. đường tròn $(K)$ qua $B$ và $C$ cắt $AC$ và $AB$ tại $E$ và $F.BE$ cắt $CF$ tại $G.AG$ cắt $BC$ tại $P$. Hạ $PH$ vuông góc $EF$. Đường tròn ngoại tiếp tam giác $KEF$ cắt đường trung trực $BC$ tại điểm thứ $2$ là $I$. Chứng minh $AH$ và $AI$ là $2$ đường đẳng giác $\angle A$. 




#681744 SHARE KHO GIÁO TRÌNH SSDG CỰC LỚN HOT HOT ĐA CHỦNG LOẠI

Đã gửi bởi xuantrandong on 24-05-2017 - 00:55 trong Phần mềm hỗ trợ học tập, giảng dạy - Các trang web hay

kho ebook ssdg

http://4share.vn/d/6257575252535653




#633647 Chứng minh $AS$ song song với $BC$

Đã gửi bởi xuantrandong on 17-05-2016 - 15:00 trong Hình học

cho 2 đường tròn $\odot(O)$ và $\odot(O')$ không cắt nhau . gọi $d$ là trục đẳng phương của 2 đường tròn này. 1 đường thẳng bất kì cắt  $\odot(O)$ và $\odot(O')$ lần lượt tại $X,B,Y,C$, $X,B\in (O); Y,C \in (O')$. Gọi $A$ và $T$ là 2 điểm bất kì thuộc $d$. Đường thẳng bất kì qua $T$ cắt $AB,AC$ lần lượt tại $E,F$. Đường thẳng bất kì qua $T$ cắt $AX,AY$ lần lượt tại $G,H$. Gọi $S$ là giao điểm của $GF$ và $HE$. Chứng minh $AS$ song song với $BC$

 

 

 

 

cách giải của mình k hay. Mọi người góp ý nha